4.2 对数函数

\({\color{Red}{欢迎到学科网下载资料学习 }}\)
[ 【高分突破系列】高一数学上学期同步知识点剖析精品讲义与分层练习]
(https://www.zxxk.com/docpack/2783085.html)
\({\color{Red}{ 跟贵哥学数学,so \quad easy!}}\)

必修第一册同步拔高练习,难度3颗星!

模块导图

知识剖析

对数的概念

① 概念
一般地,如果\(a^x=N\)(\(a>0\),且\(a≠1\)),那么数\(x\)叫做以\(a\)为底\(N\)的对数,记作\(x=log_a N\).
(\(a\)底数,\(N\)真数,\(log_a N\)对数)
② 两个重要对数
常用对数以\(10\)为底的对数,\(\log_{10}N\)记为\(\lg N\)
自然 对数以无理数\(e\)为底的对数的对数,\(\log_e N\)记为\(\ln N\)
③ 对数式与指数式的互化

 

④ 结论
\((1)\)负数和零没有对数
\((2)\log_a a=1\)\(\log_a 1=0\).
特别地,\(\lg 10=1\)\(\lg 1=0\)\(\ln e=1\)\(\ln 1=0\).

对数的运算

如果\(a>0\)\(a ≠ 1\),\(M>0\),\(N>0\), 有
\(\log _{a}(M N)=\log _{a} M+\log _{a} N\)
\(\log _{a} \dfrac{M}{N}=\log _{a} M-\log _{a} N\)
\(\log _{a} M^{n}=n \log _{a} M(n \in R)\)
\(a^{\log _{a} M}=M\)
⑤ 换底公式
\(\log _{a} b=\dfrac{\log _{c} b}{\log _{c} a}(a>0, a \neq 1, c>0, c \neq 1, b>0)\)
利用换底公式推导下面的结论
\(\log _{a} b=\dfrac{1}{\log _{b} a}\)
\(\log _{a} b \cdot \log _{b} c=\log _{a} c\)
\(\log _{a^{m}} b^{n}=\dfrac{n}{m} \log _{a} b\)
特别注意:\(\log _{a} M N \neq \log _{a} M \cdot \log _{a} N\)
\(\log _{a}(M \pm N) \neq \log _{a} M \pm \log _{a} N\)
 

对数函数

① 对数函数的概念
函数\(y=log_a x(a>0 ,a ≠1)\)叫做对数函数,其中\(x\)是自变量.
② 图像与性质

函数名称 对数函数
定义 函数$y=log_a x(a>0 ,a ≠1)$叫做对数函数
图象 $a>1$ $0< a <1$
定义域 $(0,+∞)$
值域 $R$
过定点 图象过定点$(1 ,0)$
奇偶性 非奇非偶
单调性 在$(0 ,+∞)$上是增函数 在$(0 ,+∞)$上是减函数
$a$变化对图象的影响 在第一象限内,$α$越大图象越靠低; 在第四象限内,$α$越大图象越靠高.

 

经典例题

【题型一】对数的化简与求值

【典题1】求值\(2 \log _{3} 2-\log _{3} \dfrac{32}{9}+\log _{3} 8-5^{\log _{5} 3}+(\lg 5)^{2}+\lg 2 \times \lg 50\)
【解析】\(2 \log _{3} 2-\log _{3} \dfrac{32}{9}+\log _{3} 8-5^{\log _{5} 3}+(\lg 5)^{2}+\lg 2 \times \lg 50\)
\(=\log _{3} 4-\log _{3} \dfrac{32}{9}+\log _{3} 8-3+(\lg 5)^{2}+2 \lg 2 \cdot \lg 5+(\lg 2)^{2}\)
\(=\log _{3}\left(4 \times \dfrac{9}{32} \times 8\right)-3+(\lg 5+\lg 2)^{2}\)
\(=2-3+1\)
\(=0\)
 

【典题2】\(x\),\(y\),\(z∈R^+\),且\(3^x=4^y=12^z\)\(\dfrac{x+y}{z} \in(n, n+1)\)\(n∈N\),则\(n\)的值是\(\underline{\quad \quad }\).
【解析】\(3^x=4^y=12^z=k>1\)
\(x=\log _{3} k=\dfrac{l g k}{l g 3}\)\(y=\log _{4} k=\dfrac{l g k}{\lg 4}\)\(z=\log _{12} k=\dfrac{l g k}{l g 12}\)
\({\color{Red}{(利用换底公式,把数值化为同底,有利于\dfrac{x+y}{z}求值去掉k)}}\)
\(\therefore \dfrac{x+y}{z}=\dfrac{\dfrac{\lg k}{\lg 3}+\dfrac{\lg k}{\lg 4}}{\dfrac{\lg k}{\lg 12}}=\dfrac{\lg 12 \cdot \lg 12}{\lg 3 \cdot \lg 4}\)\(=\dfrac{(\lg 3+\lg 4)^{2}}{\lg 3 \cdot \lg 4}=\dfrac{\lg 3}{\lg 4}+\dfrac{\lg 4}{\lg 3}+2\)
\({\color{Red}{(∵\dfrac{x+y}{z} \in(n, n+1),∴要对\dfrac{\lg 3}{\lg 4}+\dfrac{\lg 4}{\lg 3}+2进行估值,要把其值的整数部分求出)}}\)
\(\because 0<\dfrac{\lg 3}{\lg 4}<1\)
\(\therefore \dfrac{\lg 3}{\lg 4}+\dfrac{\lg 4}{\lg 3}>2\)\({\color{Red}{(利用对勾函数可得) }}\)
\(\therefore \dfrac{\lg 3}{\lg 4}+\dfrac{\lg 4}{\lg 3}+2>4\)
\(\because \dfrac{\lg 4}{\lg 3}<2\),\(\dfrac{\lg 3}{\lg 4}<1\)\(\therefore \dfrac{\lg 3}{\lg 4}+\dfrac{\lg 4}{\lg 3}+2<5\),
\(x=\dfrac{\lg 3}{\lg 4}+\dfrac{\lg 4}{\lg 3}+2 \in(4,5)=(n, n+1)\)
\(n=4\)

巩固练习

1(★)已知函数\(f(x)=\left\{\begin{array}{l} 3^{x}(x \leq 0) \\ \log _{2} x,(x>0) \end{array}\right.\),则\(f\left[f\left(\dfrac{1}{2}\right)\right]=\) \(\underline{\quad \quad }\).
 

2(★)\((\lg 2)^{2}+\lg 5 \times \lg 20+(\sqrt{2016})^{0}+0.027^{-\frac{2}{3}} \times\left(\frac{1}{3}\right)^{-2}=\)\(\underline{\quad \quad }\)
 

3(★★)求值\(\dfrac{\lg 8+\lg 125-\lg 2-\lg 5}{\lg \sqrt{10} \cdot \lg 0.1}=\)\(\underline{\quad \quad }\)
 

4(★★)求值\(2^{\log _{2} \frac{1}{4}}-\left(\dfrac{8}{27}\right)^{-\frac{2}{3}}+\lg \dfrac{1}{100}+(\sqrt{2}-1)^{\lg 1}=\)\(\underline{\quad \quad }\)
 

5(★★)\(a>1\)\(b>1\)\(\lg \left(1+\dfrac{b}{a}\right)=\lg b\),则\(\lg (a-1)+\lg (b-1)\)的值\(\underline{\quad \quad }\)
 

6(★★★)已知\(2^a=7^b=m\)\(\dfrac{1}{a}+\dfrac{1}{2 b}=\dfrac{1}{2}\),则\(m=\) \(\underline{\quad \quad }\).
 

7(★★★)已知\(a>b>1\),若\(\log _{a} b+\log _{b} a=\dfrac{5}{2}\)\(a^b=b^a\),则\(ab=\) \(\underline{\quad \quad }\)
 

参考答案

1.\(\dfrac{1}{3}\)
2.\(102\)
3.\(-4\)
4.\(-3\)
5.\(0\)
6.\(28\)
7.\(8\)
 

【题型二】对数函数的图象及应用

【典题1】函数\(y=\log _{a}(|x|+1)(a>1)\)的图象大致是(  )

【解析】 \({\color{Red}{方法1}}\)
\(y=\log _{a}(|x|+1)=\left\{\begin{array}{c} \log _{a}(x+1), x \geq 0 \\ \log _{a}(-x+1), x<0 \end{array}\right.\)
\(a>1\),由对数函数的性质易得选\(B\).
\({\color{Red}{方法2\quad 函数图象变换}}\)

故选\(B\)
【点拨】涉及对数函数型的函数\(y=f(x)\),往往需要得到其图象,方法有
① 利用要相应指数函数的图象通过平移、对称、翻转变换得其图象;
② 利用去掉绝对值得到分段函数得其图象.
 

【典题2】\(a\),\(b\)\(c\)均为正数,且\(2^{a}=\log _{\frac{1}{2}} a\)\(\left(\dfrac{1}{2}\right)^{b}=\log _{\frac{1}{2}} b\)\(\left(\dfrac{1}{2}\right)^{c}=\log _{2} c\),则(  )
A.\(a<b<c\) \(\qquad \qquad\) B.\(c<b<a\) \(\qquad \qquad\)C.\(c<a<b\) \(\qquad \qquad\) D.\(b<a<c\)
【解析】分别作出四个函数\(y=\left(\dfrac{1}{2}\right)^{x}\),\(y=\log _{\frac{1}{2}} x\)\(y=2^{x}\)\(y=\log _{2} x\)的图象,观察它们的交点情况.由图象知\(a<b<c\).故选\(A\)

【点拨】
\(2^{a}=\log _{\frac{1}{2}} a\)\(a\)是函数\(y=2^x\)\(y=\log _{\frac{1}{2}} x\)的交点横坐标;
② 函数\(y=2^x\)\(y=\log_2⁡x\)互为反函数,图象关于直线\(y=x\)对称. 函数\(y=\left(\frac{1}{2}\right)^{x}\)\(y=\log _{\frac{1}{2}} x\)也是.
 

【典题3】已知\(f(x)=\left\{\begin{array}{l} 3\left|\log _{3} x\right|, 0<x \leq 3 \\ (x-4)(x-6), x>3 \end{array}\right.\),若\(f(a)=f(b)=f(c)=f(d)\),且\(a<b<c<d\),则\(abcd\)的取值范围是\(\underline{\quad \quad }\).
\({\color{Red}{思考痕迹}}\) 已知条件\(f(a)=f(b)=f(c)=f(d)\),相当于\(y=f(x)\)与一直线\(y=k\)相交于四个点,四点的横坐标是\(a、b、c、d\),所以想到数形结合.
【解析】先画出\(f(x)=\left\{\begin{array}{l} 3\left|\log _{3} x\right|, 0<x \leq 3 \\ (x-4)(x-6), x>3 \end{array}\right.\)的图象,如图

\(∵a ,b ,c ,d\)互不相同,不妨设\(a<b<c<d\)
\(f(a)=f(b)=f(c)=f(d)\)\(3<c<4\)
由图可知\(|\log_3⁡a |=|\log_3⁡b |\)\(c、d\)关于\(x=5\)对称,
\(∴-\log_3⁡a=\log_3⁡b\)\(c+d=10\)
\(ab=1\),\(c+d=10\)
\(a b c d=c(10-c)=-(c-5)^{2}+25\)
由图象可知\(3<c<4\)
由二次函数的知识可知\(21<-c^2+12c<24\)
\(∴abcd\)的范围为\((21 ,24)\)
【点拨】遇到分段函数,经常用数形结合的方法画出函数图象,注意一些关键的临界值,比如\(x=3\)处.
 

巩固练习

1(★)已知\(lga+lgb=0\),函数\(f(x)=a^x\)与函数\(g(x)=-log_b⁡x\)的图象可能是(  )

 

2(★)已知图中曲线\(C_1\),\(C_2\),\(C_3\),\(C_4\)分别是函数\(y=\log _{a_{1}} x\)\(y=\log _{a_{2}}x\)\(y=\log _{a_{3}} x\)\(y=\log _{a_{4}} x\)的图象,则\(a_1\),\(a_2\),\(a_3\),\(a_4\)的大小关系是(  )

A.\(a_4<a_3<a_2<a_1\)\(\qquad \qquad \qquad \qquad\) B.\(a_3<a_4<a_1<a_2\)
C.\(a_2<a_1<a_3<a_4\)\(\qquad \qquad \qquad \qquad\) D.\(a_3<a_4<a_2<a_1\)
 

3(★★)已知函数\(f(x)=|\ln x|\),若\(0<a<b\),且\(f(a)=f(b)\),则\(a+5b\)的取值范围是(  )
A.\((2 \sqrt{5},+\infty)\)\(\qquad \qquad\)B.\([2 \sqrt{5},+\infty)\)\(\qquad \qquad\)C.\((6 ,+∞)\) \(\qquad \qquad\)D.\([6 ,+∞)\)

 

4(★★)已知函数\(f(x)=\left|\log _{a}\right| x-1||\)\((a>0, a \neq 1)\),若\(x_1<x_2<x_3<x_4\),\(x_1 x_2 x_3 x_4≠0\)\(f(x_1)=f(x_2)=f(x_3)=f(x_4)\),则\(x_1+x_2+x_3+x_4=\)(  )
A.\(2\) \(\qquad \qquad \qquad \qquad\) B.\(4\)\(\qquad\qquad \qquad \qquad\)C.\(8\) \(\qquad \qquad \qquad\qquad\)D.随\(a\)值变化
 

5(★★★)已知函数\(f(x)=\left|\log _{2}(x-1)\right|\)\(g(x)=\left(\dfrac{1}{2}\right)^{x}\),则图象交于\(A(x_1 ,y_1)\)\(B(x_2 ,y_2)\)两点,则(  )
A.\(x_1 x_2<1\)\(\qquad \qquad\)B.\(x_1+x_2>5\)\(\qquad \qquad\)C.\(x_1+x_2>x_1 x_2\) \(\qquad \qquad\)D.\(x_1+x_2<x_1 x_2\)
 

6(★★★)已知函数\(f(x)=\left\{\begin{array}{l} \left|\log _{2} x\right|, 0<x \leq 8 \\ -\dfrac{1}{4} x+5, x>8 \end{array}\right.\),若\(a ,b ,c\)互不相等,且\(f(a)=f(b)=f(c)\),则\(abc\)的取值范围是\(\underline{\quad \quad }\)
 

7(★★★)已知函数\(f(x)=|\log_2⁡x |,\)\(g(x)=\dfrac{1}{2} x\),若对任意\(x∈[a ,+∞)\),总存在两个\(x_{0} \in\left[\dfrac{1}{2}, 4\right]\),使得\(g(x)\cdot f(x_0)=1\),则实数\(a\)的取值范围是\(\underline{\quad \quad }\)
 
 
 

参考答案

1.\(B\)
2.\(B\)
3.\(C\)
4.\(B\)
5.\(C\)
6.\((8 ,20)\)
7.\([2 ,+∞)\)

【题型三】对数函数的性质及应用

角度1 比较对数式的大小

【典题1】已知\(a=\log _{2} 7\)\(b=\log _{3} 8\)\(c=0.3^{0.2}\),则\(a\)\(b\)\(c\)的大小关系为(  )
A.\(c<b<a\) \(\qquad \qquad\)B.\(a<b<c\) \(\qquad \qquad\)C.\(b<c<a\) \(\qquad \qquad\)D.\(c<a<b\)
【解析】由题意,可知\(a=\log_2⁡7>\log_2⁡4=2\)
\(c=0.3^{0.2}<0.3^{0}=1\)
\(\because 1<\log _{3} 8<\log _{3} 9=2\)
\(\therefore 1<b<2\)
\(∴c<b<a\)
故选\(A\)
 

【典题2】\(a=\log _{2} 3\)\(b=\dfrac{4}{3}\)\(c=\log _{3} 4\),则\(a\),\(b\),\(c\)的大小关系为(  )
A.\(b<a<c\)\(\qquad \qquad\)B.\(c<a<b\) \(\qquad \qquad\) C.\(a<b<c\) \(\qquad \qquad\) D.\(c<b<a\)
【解析】\(\because a=\log _{2} 3>\log _{2} 2^{\frac{4}{3}}=\dfrac{4}{3}=b\)\(b=\dfrac{4}{3}=\log _{3} 3^{\frac{4}{3}}>\log _{3} 4=c\)
\(∴a ,b ,c\)的大小关系为\(c<b<a\)
故选\(D\)
 

【典题3】已知\(a=\log _{5} 2\)\(b=\log _{0.5} 0.2\)\(c=0.5^{0.2}\),则\(a\),\(b\),\(c\)的大小关系为(  )
A.\(a<c<b\) \(\qquad \qquad\)B.\(a<b<c\) \(\qquad \qquad\)C.\(b<c<a\) \(\qquad \qquad\)D.\(c<a<b\)
【解析】由题意,可知\(a=\log _{5} 2<1\)\(c=0.5^{0.2}<1\)
\(b=\log _{0.5} 0.2=\log _{\frac{1}{2}} \frac{1}{5}=\log _{2} 5>\log _{2} 4=2\)
\({\color{Red}{(初步估值) }}\)
\(∴b\)最大,\(a、c\)都小于\(1\)
\({\color{Red}{(b,c还比较不出来,进一步估值) }}\)
\(\because a=\log _{5} 2=\dfrac{1}{\log _{2} 5}<\dfrac{1}{2}\)\(c=0.5^{0.2}=\left(\dfrac{1}{2}\right)^{\frac{1}{5}}=\sqrt[5]{\dfrac{1}{2}}>\dfrac{1}{2}\)
\(∴a<c\)\({\color{Red}{(引入第三数\dfrac{1}{2}比较)}}\)
\(∴a<c<b\),故选:\(A\)
【点拨】比较对数的大小,主要是利用对数函数的单调性,具体方法有
① 把对数化为同底,利用对数函数的单调性比较大小;
② 若不能化为同底,可对对数进行估值,一般可以与\(0\)\(1\)比较大小;
③ 利用第三个数作为两个数字大小比较的过渡.
 

角度2 求解对数型不等式和方程

【典题1】方程\(\log _{2}(x-1)=2-\log _{2}(x+1)\)的解集为\(\underline{\quad \quad }\)
【解析】\(∵\log _{2}(x-1)=2-\log _{2}(x+1)\)
\(∴\log _{2}(x-1)=\log _{2} \dfrac{4}{x+1}\)
\(\therefore x-1=\dfrac{4}{x+1}\),解得\(x=\pm \sqrt{5}\)
检验得\(x=-\sqrt{5}\)不符合, \({\color{Red}{ (注意真数的范围) }}\)
\(∴\)方程\(\log _{2}(x-1)=2-\log _{2}(x+1)\)的解集为\(\{\sqrt{5}\}\)
故答案为\(\{\sqrt{5}\}\)
 

【典题2】不等式\(\log _{2}\left(x^{2}-1\right)<3\)的解集为\(\underline{\quad \quad }\).
【解析】\(\log _{2}\left(x^{2}-1\right)<3\)\(\Leftrightarrow \log _{2}\left(x^{2}-1\right)<\log _{2} 8\)
\(∴0<x^2-1<8\) \({\color{Red}{(误解x^2-1<8) }}\)
解得\(-3<x<-1\)\(1<x<3\).
【点拨】在处理对数的方程和不等式时不要忘记了“对数\(log_a x\)中真数\(x>0\)”这点.
 

角度3 对数型函数综合问题

【典题1】函数\(y=\log _{\frac{1}{2}}\left(x^{2}-6 x+17\right)\)的值域是\(\underline{\quad \quad }\) .
【解析】\(∵t=x^2-6x+17=(x-3)^2+8≥8\)
\(∴\)内层函数的值域\([8 ,+∞),\)
\(y=\log _{\dfrac{1}{2}} t\)\([8 ,+∞)\)是减函数,
\(y \leq \log _{\dfrac{1}{2}} 8=-3\)
\(∴\)函数\(y=\log _{\frac{1}{2}}\left(x^{2}-6 x+17\right)\)的值域是\((-∞ ,-3]\).
【点拨】复合函数的值域先求内层函数值域再求外层函数.
 

【典题2】已知函数\(f(x)\)\(R\)上的奇函数,且满足\(f(x+2)=-f(x)\),当\(x∈(0 ,1]\)时,\(f(x)=2^x-1\),则方程\(f(x)=\log _{7}|x-2|\)解的个数是\(\underline{\quad \quad }\).
【解析】函数\(f(x)\)\(R\)上的奇函数,\(f(0)=0\)
\(f(x+2)=-f(x)\),可得\(f(x+2)=f(-x)\)
\(∴f(x)\)的有条对称轴\(x=1\)
\(f(x+2)=-f(x)\),可得\(f(x+4)=f(x)\)
\(∴f(x)\)的周期\(T=4\)
(注 由以上已知,较容易画出\(y=f(x)\)的图象,作图步骤如下
① 画\(f(x)=2^x-1 ,x∈(0 ,1)\)

image.png
② 根据奇函数的性质

image.png
③ 由对称轴\(x=1\)可得
image.png
④ 由周期\(T=4\)可得

作出在同一坐标系中画\(y=f(x)\)\(g(x)=\log _{7}|x-2|\)图象,

注意到\(g(9)=1\),\(g(-7)>1\)\({\color{Red}{(注意一些临界的位置) }}\)
从图象不难看出,其交点个数\(7\)个.
【点拨】
① 遇到函数综合性质问题(有单调性,对称性,周期性等),一般通过数形结合的方法处理;
\((1)f(x+a)=f(x+b)\)\(⇒f(x)\)的周期\(T=a-b\)
\((2)f(x+a)=f(b-x)\)\(⇒f(x)\)的对称轴\(x=\dfrac{a+b}{2}\)
\((3)f(x+a)=-f(x)\)\(⇒f(x)\)的周期\(T=2a\)
\((4)f(x+a)=\dfrac{1}{f(x)}\)\(⇒f(x)\)的周期\(T=2a\).
 

【典题3】\(a>0\)\(b>0\),则下列叙述正确的是(  )
A.若\(\ln a-2 b>\ln b-2 a\),则\(a>b\) \(\qquad \qquad\) B.若\(\ln a-2 b>\ln b-2 a\),则\(a<b\)
C.若\(\ln a-2 a>\ln b-2 b\),则\(a>b\)\(\qquad \qquad\)D.若\(\ln a-2 a>\ln b-2 b\),则\(a<b\)
【解析】 \({\color{Red}{方法1 \quad构造函数法}}\)
\(∵y=\ln x\)\(y=2x\)均为增函数,
\(f(x)=\ln x+2x\)\((0 ,+∞)\)上为增函数,
\(f(a)>f(b)⇔a>b>0\)
\(\ln a+2 a>\ln b+2 b \Leftrightarrow a>b>0\)
\(\ln a-2 b>\ln b-2 a \Leftrightarrow a>b>0\)
故选\(A\)
\({\color{Red}{方法2\quad 取特殊值排除法}}\)
对于\(A、B\)
\(a=1\)\(b=\dfrac{1}{e}\)
代入\(\ln a-2b>\ln b-2a\)\(-\dfrac{2}{e}>-3\)显然成立,
\(a>b\),此时可排除选项\(B\)
对于选项\(C、D\)
\(a=1\)\(b=e\),代入\(\ln a-2 a>\ln b-2 b\)
\(-2>1-2e\)显然成立,而\(a<b\)可排除选项\(C\)
\(a=1\)\(b=\dfrac{1}{e^{2}}\),代入\(\ln a-2 a>\ln b-2 b\)
\(-2>-2-\dfrac{2}{e^{2}}\)显然成立,而\(a>b\)可排除选项\(D\)
故选A.
【点拨】
① 方法1通过构造函数\(f(x)=\ln x+2x\),利用其单调性进行选项判断.构造函数的方法到了高二还经常见,可以先熟悉先!
② 方法2“取特殊值排除法”,在取数时一定要满足题目要求,尽量取容易计算的数值,要大胆尝试,能排除一个是一个.
 

【典题4】已知函数\(f(x)=\log _{3} \dfrac{1-x}{1+x}\)
(1)求函数\(f(x)\)的定义域;
(2)判断函数\(f(x)\)的奇偶性;
(3)当\(x \in\left[-\dfrac{1}{2}, \dfrac{1}{2}\right]\)时,函数\(g(x)=f(x)\),求函数\(g(x)\)的值域.
【解析】(1)要使函数\(f(x)=\log _{3} \dfrac{1-x}{1+x}\)的解析式有意义,
自变量\(x\)须满足\(\dfrac{1-x}{1+x}>0\),解得\(x∈(-1 ,1)\)
故函数\(f(x)\)的定义域为\((-1 ,1)\)
(2)由(1)得函数的定义域关于原点对称,
\(f(-x)=\log _{3} \dfrac{1+x}{1-x}=-\log _{3} \dfrac{1-x}{1+x}=-f(x)\)
故函数\(f(x)\)为奇函数;
(3)当\(x \in\left[-\dfrac{1}{2}, \dfrac{1}{2}\right]\)时,
\(u(x)=\dfrac{1-x}{1+x}=\dfrac{2}{1+x}-1\) \({\color{Red}{(分离常数法)}}\)
\({\color{Red}{(注 函数图象如右图,由y=\dfrac{2}{x}向左向下平移一个单位得到的) }}\)
\(u(x)=\dfrac{1-x}{1+x}\)\(\left[-\dfrac{1}{2}, \dfrac{1}{2}\right]\)上为减函数,
\(u(x) \in\left[\dfrac{1}{3}, 3\right]\)
\(\because g(x)=f(x)=\log _{3} u\)为增函数,
\(g(x)∈[-1 ,1]\)
故函数\(g(x)\)的值域为\([-1 ,1]\)
【点拨】
① 遇到形如\(f(x)=\dfrac{a \cdot g(x)+b}{c \cdot g(x)+d}\)的函数(比如\(y=\dfrac{1-2 x}{1+x}\)\(y=\dfrac{2^{x}-3}{2^{x}+4}\)\(y=\dfrac{3 x^{2}+4}{x^{2}-1}\)等)均可采取“分离常数法”,易求函数的单调性,对称性,最值等性质;
② 求复合函数的值域,要分清楚内层函数与外层函数,分别对它们的单调性进行分析再求值域,函数的定义域优先考虑.
 

【典题5】\(D\)是函数\(y=f(x)\)定义域的一个子集,若存在\(x_0∈D\),使得\(f(x_0 )=-x_0\)成立,则称\(x_0\)\(f(x)\)的一个“准不动点”,也称\(f(x)\)在区间\(D\)上存在准不动点.
已知\(f(x)=\log _{\frac{1}{2}}\left(4^{x}+a \cdot 2^{x}-1\right)\)\(x \in[0,1]\)
(1)若\(a=1\),求函数\(f(x)\)的准不动点;
(2)若函数\(f(x)\)在区间\([0 ,1]\)上存在准不动点,求实数\(a\)的取值范围.
【解析】(1)当\(a=1\)时,
可得\(f(x)=\log _{\frac{1}{2}}\left(4^{x}+2^{x}-1\right)=-x\)\(x∈[0 ,1]\)
可得\(4^x+2^x-1=2^x\)
\(4^x=1\)\(∴x=0\)
\(a=1\),函数\(f(x)\)的准不动点为\(x_0=0\)
(2) \({\color{Red}{方法1}}\) 由定义可得
方程\(\log _{\frac{1}{2}}\left(4^{x}+a \cdot 2^{x}-1\right)=-x\)\(x∈[0 ,1]\)上有解
即方程\(4^x+a⋅2^x-1=2^x\)\(x∈[0 ,1]\)上有解,
\(4^x+a\cdot 2^x-1>0\)\((*)\)
\(2^x=t\)\(x∈[0 ,1]\),则\(t∈[1 ,2]\)
那问题\((*)\)转化为方程\(t^2+(a-1)t-1=0\)\([1 ,2]\)有解,且\(t^2+at-1>0\)
\(g(t)=t^2+(a-1)t-1\),开口向上且\(g(0)<0\)
所以\(y=g(t)\)\([1 ,2]\)上与\(x\)轴只有一个交点,
则只需要\(g(1)g(2)≤0\),解得\(-\dfrac{1}{2} \leq a \leq 1\)
\({\color{Red}{(一元二次方程根的分布问题,注意数形结合分析)}}\)
要使\(t^2+at-1>0(1≤t≤2)\)恒成立.
其对称轴\(x=-\dfrac{a}{2}\),在\(1≤t≤2\)上是递增的,
\(t=1\)时最小值,可得\(a>0\)
综上可得实数\(a\)的取值范围是\((0,1]\)
\({\color{Red}{方法2 }}\)
与方法1同样得到方程\(t^2+(a-1)t-1=0\)\([1 ,2]\)有解,且\(t^2+at-1>0\)
\(a=1-t+\dfrac{1}{t}\)\(t∈[1 ,2]\)上有解,
\(a>\dfrac{1}{t}-t\)\(t∈[1 ,2]\)上恒成立 \({\color{Red}{(分离参数法)}}\)
\(h(t)=1-t+\dfrac{1}{t}\)\(t∈[1 ,2]\)上显然是减函数,
其值域为\(\left[-\dfrac{1}{2}, 1\right]\),则\(-\dfrac{1}{2} \leq a \leq 1\)
\(d(t)=\dfrac{1}{t}-t\)\(t∈[1 ,2]\)上显然是减函数,最大值为\(d(1)=0\),则\(a>0\),
综上可得实数\(a\)的取值范围是\((0,1]\)
【点拨】
① 在第二问中不要漏了\(4^x+a\cdot 2^x-1>0\),求解过程中谨记等价转化,做到严谨;
② 第二问的方法1是采取了“二次方程根的分布问题”的处理技巧,注意结合二次函数图象进行思考;方法2是采取分离参数法转而求最值,
 

巩固练习

1(★)\(a=\log _{2} 1.5\),\(b=\log _{2} 0.1\),\(c=2^{0.2}\),则(  )
A.\(c<b<a\) \(\qquad \qquad\)B.\(b<c<a\) \(\qquad \qquad\)C.\(a<b<c\) \(\qquad \qquad\)D.\(b<a<c\)
 

2(★★)\(a=\log _{\frac{1}{2}} 6\)\(b=\log _{\frac{1}{4}} 12\)\(c=\log _{\frac{1}{5}} 15\),则(  )
A.\(a<b<c\) \(\qquad \qquad\)B.\(c<b<a\) \(\qquad \qquad\)C.\(b<a<c\) \(\qquad \qquad\)D.\(c<a<b\)
 

3(★★)\(f(x)\)是定义在\(R\)上的函数,且\(f(2-x)=f(x)\),当\(x≥1\)时,\(f(x)=\log_2⁡x\),则有(  )
A.\(f\left(\dfrac{1}{3}\right)<f(2)<f\left(\dfrac{1}{2}\right)\)\(\qquad \qquad\)B.\(f\left(\dfrac{1}{2}\right)<f(2)<f\left(\dfrac{1}{3}\right)\)
C.\(f\left(\dfrac{1}{2}\right)<f\left(\dfrac{1}{3}\right)<f(2)\)\(\qquad \qquad\)D.\(f(2)<f\left(\dfrac{1}{2}\right)<f\left(\dfrac{1}{3}\right)\)
 

4(★★)不等式\(\log _{2}\left(2^{x}-1\right) \cdot \log _{2}\left(2^{x+1}-2\right)<2\)的解集为\(\underline{\quad \quad }\)
 

5(★★)函数\(f(x)=\log _{\frac{1}{3}}\left(x^{2}-3 x+2\right)\)的单调递增区间为\(\underline{\quad \quad }\)
 

6(★★)方程\(\log _{2}\left(4^{x}-3\right)=x+1\)的解集为\(\underline{\quad \quad }\)
 

7(★★★)已知函数\(f(x)=\log _{a}(x+1)\)\(g(x)=2 \log _{a}(2 x+t)(t \in R)\)\(a>0\),且\(a≠1\)
(1)若\(1\)是关于\(x\)的方程\(f(x)-g(x)=0\)的一个解,求\(t\)的值;
(2)当\(0<a<1\)\(t=-1\)时,解不等式\(f(x)≤g(x)\)
(3)若函数\(F(x)=a^{f(x)} +tx^2-2t+1\)在区间\((-1 ,2]\)上有零点,求\(t\)的取值范围.
 
 
 

参考答案

1.\(D\)
2.\(A\)
3.\(C\)
4.\(\left(\log _{2} \dfrac{5}{4}, \log _{2} 3\right)\)
5.\((-∞ ,1)\)
6.\(\{log_23\}\)
7.\(\text { (1) } t=\sqrt{2}-2\)\(\text { (2) } \dfrac{1}{2}<x \leq \dfrac{5}{4}\)\(\text { (3) } t \leq-2 \text { 或 } t \geq \dfrac{2+\sqrt{2}}{4}\)

posted @ 2022-01-05 10:23  贵哥讲数学  阅读(1069)  评论(0编辑  收藏  举报
//更改网页ico // 实现数学符号与汉字间有间隙 //文章页加大页面,隐藏侧边栏